LSAT and Law School Admissions Forum

Get expert LSAT preparation and law school admissions advice from PowerScore Test Preparation.

 htngo12
  • Posts: 40
  • Joined: May 19, 2016
|
#28802
Hi!

For this assumption question, I was able to gather what the stimulus was saying after reading a few times.

However, I'm still a little confused on how to set up the argument.

Here's my current thought process:

P: Totalitarian regimes influential (sufficient) ->reduce public passivity (necessary)

C: Political regimes censor forms of expression that threaten their power.

I processed ~(reduce passivity) then ~(jnfluential) then ~(threaten power) to get to my answer (C).

I am conflicted whether the premise should have been set up as:

1) Totalitarian regimes influential (suff) -> blasphemous or pornographic (necc)

2) Line 1 above (cause) -> reduce public positivity (effect)

Does the premise contain a conditional statement, causal relationship, or both?
User avatar
 Jonathan Evans
PowerScore Staff
  • PowerScore Staff
  • Posts: 726
  • Joined: Jun 09, 2016
|
#28930
Hi, htngo12,

It seems as though your approach to this stimulus is slightly overwrought. Remember that there is great value in simplicity, and the more precise and concise you can be with your analysis, the faster and more accurately you will find the credited response. It is not uncommon for the LSAT writers to make concepts or ideas in the premises somewhat abstruse or byzantine in an effort to confound your ability to remark the most important elements.

You could diagram this stimulus as follows:

Political regime that censors :arrow: censors perceived threat to power

Conclusion: Political regime that censors :arrow: censors material that decreases public passivity

Notice that your premises and conclusion are reversed. The author assumes that material that decreases public passivity implies a perceived threat to power. Also, I have kept my terms as consistent as possible between the premises and conclusion to avoid introducing sources of confusion or ambiguity. Likewise, I dispense with the irrelevant parts of the stimulus (those that do not help better understand the underlying structure). Finally, I would not necessarily symbolize such a problem on the test as such work could in fact confuse the issue for you. Your goal should be perhaps to talk yourself through a problem like this with the intention of discerning the gap in the reasoning without resorting to a possibly erroneous symbolic representation of the logic involved.

Symbolization is a powerful tool but is a complement to and not a substitute for your own analysis of the reasoning contained in a stimulus. You must give a concise and accurate description of the reasoning in a stimulus before you symbolize it, and in a case such as this, it might be better not to symbolize at all when actually taking the test.
 htngo12
  • Posts: 40
  • Joined: May 19, 2016
|
#34422
Hi!
I do apologize for not getting back on this post. With more LSAT testing experience, I have been able to improve my stimulus analysis skills. Instead of being more of an analytical thinker, I am looking at the arguments as a whole.
For this argument, finding the conclusion was initially confusing, but I used the therefore method.
I do find that the language can be difficult to simplify at times. For example, linking totalitarian regime with political regimes. However, I can see the linkage if I reread the stimulus. The premise has to connect with the conclusion.
So for the conclusion to be true, the historian assumes that totalitarian regimes associate reducing passivity with threatening of power.
Thanks for your help!
 ShannonOh22
  • Posts: 70
  • Joined: Aug 15, 2019
|
#72108
Hi PowerScore,

Can you please elaborate on why E is incorrect? E reads: "Most writings that totalitarian regimes label blasphemous or pornographic would, if widely influential, reduce public passivity." I know this is nearly the exact same statement as the last sentence, but it changes it in a small yet significant way - it confirms the fact that the writings the totalitarian regime classifies as "blasphemous or pornographic" WOULD in fact reduce public passivity.

I see how C is correct, but I'm having trouble explaining why C is a better answer than E. Any help is greatly appreciated!
User avatar
 KelseyWoods
PowerScore Staff
  • PowerScore Staff
  • Posts: 1079
  • Joined: Jun 26, 2013
|
#72324
Hi Shannon!

The shift in answer choice (E) is not that it confirms that the writings the totalitarian regime classifies as blasphemous or pornographic would in fact reduce public passivity--the last sentence of the stimulus already confirmed that was the case. The last sentence tells us that if writings would reduce public passivity, totalitarian regimes classify them as blasphemous or pornographic. The shift in (E) is that is says that MOST of the writings labeled as blasphemous or pornographic would reduce public passivity.

A diagram might help with conceptualizing this:

Conclusion:
Reduce public passivity :arrow: Labeled as blasphemous/pornographic
From this we can infer:
Labeled as blasphemous/pornographic :some: Reduce public passivity

Answer choice (E)
Labeled as blasphemous/pornographic :most: Reduce public passivity

So the shift is really more from the SOME to the MOST. This shift is not something that is necessary for our conclusion. We are looking for something that is necessary for the conclusion that if a writing would reduce public passivity, then it will be censored. It's not necessary that MOST of the writings that are censored would reduce public passivity.

Answer choice (C) is necessary to this argument because it links our conclusion to the premises (loss of public passivity = threat to power).

Hope this helps!

Best,
Kelsey
 Legallyconfused
  • Posts: 19
  • Joined: Oct 03, 2019
|
#72912
Hi all!

Here is why I did not initially pick AC C.

I negated C to say: A totalitarian regime CANNOT perceive the loss of public passivity as a threat to its power”. And I thought so? If it does not perceive it as a threat to its power, maybe it is censoring the writings that would reduce public passivity because the regime believes it will undermine public morality. It said in the first sentence that the regimes sometimes censor things because they believe it will undermine public morality.

Can I not connect that first sentence to the conclusion. If not, why?

Thanks in advance!
 Adam Tyson
PowerScore Staff
  • PowerScore Staff
  • Posts: 5153
  • Joined: Apr 14, 2011
|
#72984
Hey confused, Happy New Year! When negating an answer to an Assumption question, we aren't seeking to disprove the conclusion. Rather, we are looking to show that the premises no longer provide support for that conclusion. The logical force of the argument is undermined, rather than the truth of any one claim.

If totalitarian regimes cannot perceive a loss of passivity as a threat to their power, is there then any evidence to support the claim that they would label as blasphemous or pornographic such writings? Sure, they might, but where is the evidence that gets the author to this conclusion now? The only evidence was that they will do that to perceived threats, and if this stuff cannot be perceived that way, then there is nothing supporting the claim that they will censor them. It's the destruction of the force of the argument that matters, not the truth of the conclusion.

Try this example as an illustration of my point: The Panthers won the coin toss this weekend, so they will win their game next weekend.

What am I assuming? That this week's coin toss affects next week's game. What if it does not? Then the argument makes no sense. Might the Panthers still win their next game? Sure, they might, but I no longer have any evidence to back that claim up, so the argument is destroyed.

Less confused? Hope so!
 bonnie_a
  • Posts: 32
  • Joined: Jun 05, 2021
|
#89363
I got this question right but I was wondering if I could get some clarifications on the reasoning I had used to eliminate the answer choice D. I was at first tempted by this answer choice but I ended up ruling it out because I thought perhaps it's not that "widespread" public passivity is needed to retain their power. Maybe they are just concerned with the loss of or reduced passivity which they perceive to be a threat, not a "high" level of public passivity. In other words, they don't need to have widespread public passivity but rather have to maintain a certain level of passivity. Thank you in advance!
 Robert Carroll
PowerScore Staff
  • PowerScore Staff
  • Posts: 1787
  • Joined: Dec 06, 2013
|
#89866
bonnie,

That seems like a good reason to get rid of answer choice (D)! I'd only add that the perception of the regimes, not only the reality of passivity, is also the key to the stimulus, so an answer about actual and not perceived passivity is also problematic.

Robert Carroll
User avatar
 elite097
  • Posts: 18
  • Joined: Apr 09, 2023
|
#101903
WHy not B?

Get the most out of your LSAT Prep Plus subscription.

Analyze and track your performance with our Testing and Analytics Package.